LSAT and Law School Admissions Forum

Get expert LSAT preparation and law school admissions advice from PowerScore Test Preparation.

 Administrator
PowerScore Staff
  • PowerScore Staff
  • Posts: 8917
  • Joined: Feb 02, 2011
|
#47396
Complete Question Explanation
(The complete setup for this game can be found here: lsat/viewtopic.php?t=4635)

The correct answer choice is (A)

This questions asks for a list of the talks that R and S could attend together. In the correct answer to question #14, which was (C), R and S both attended F. Thus, without further consideration, answer choice (C) can be eliminated because it does not contain F. Remember—always use prior work that you are confident is correct (and with a List question you should be very confident)!
From our discussion of G, we have determined that Q must attend G. As no talk can have more than two attendees, R and S cannot both attend G. Thus, answer choice (B) can be eliminated.
Only answer choices (A), (D), and (E) remain in contention, and the difference between them is that (A) contains I and L whereas (D) does not contain I and (E) does not contain L. Thus, if you can determine whether it is possible for R and S to attend I and L together (or not), you will be able to determine the correct answer. This simple hypothetical proves that R and S can attend I together, eliminating answer choice (D):
D10_game #3_#16_diagram 1.png
In the simple hypothetical depicted above, we could switch R and S to attend L together, and have Q and T attend I together, proving that R and S can attend L together, eliminating answer choice (E).

Consequently, answer choice (A) is correct.
You do not have the required permissions to view the files attached to this post.
 BenSven
  • Posts: 3
  • Joined: Jun 20, 2020
|
#77845
I think this problem is worthy of special attention, because it exemplifies a verbal ambiguity in the prompt (which may be replicated on other questions.)

The prompt reads "Which is a complete and accurate list of the talks any one of which Rivera and Spivey could attend together?"

It's pretty clear that Rivera and Spivey can attend F, I and L together. However, they can ONLY attend L together if they have already attended F or I together. This brings us to the ambiguity: when the prompt says "any one of which" you could read that the same way you would read "any of which" (and it turns out this interpretation is correct.) But you could also read "any one of which" to restrict the number of occurrences to one. In this interpretation, choice A would be incorrect because R and S cannot attend *only* L together.

The reason I say this is worthy of attention is "any one of which" is a pretty common clause in LG question prompts. And while most games with this type of question do not have the same ambiguity, future ones might, so it's good to go into it knowing precisely what LSAC means by "any one."
User avatar
 KelseyWoods
PowerScore Staff
  • PowerScore Staff
  • Posts: 1079
  • Joined: Jun 26, 2013
|
#77904
Hi BenSven!

It's great that you are paying such close attention to the language used by the LSAT makers! As you note, it is very important to carefully read LG question stems like this to make sure you understand the restrictions imposed by the question. "Any one of which" always means the same thing as "any of which" on the LSAT. The type of restriction that you're suggesting would be indicated in a different part of the question stem.

So this question stem:

"Which one of the following is a complete and accurate list of the talks any one of which Rivera and Spivey could attend together?"

will always mean a list of any talks that R and S could attend together, regardless of what else would have to happen for them to attend that talk together.

But if the question stem had instead said:

"Which one of the following is a complete and accurate list of the talks any one of which could be the ONLY talk that Rivera and Spivey attend together?"

then you would be looking for a list of talks that R and S could attend together without attending any other talks together.

Wording on the LSAT, especially in the LG section, isn't often all that ambiguous. Which is why it's so important to read each word carefully and make sure that you aren't making any unwarranted assumptions in interpreting the rules, question stems, answer choices, etc. Rely on the words exactly as stated. And if you're ever unsure what exactly a common phrase on the LSAT means, let us know!

Hope this helps!

Best,
Kelsey
User avatar
 nkirkpatrick
  • Posts: 1
  • Joined: Jan 22, 2021
|
#83472
"From our discussion of G, we have determined that Q must attend G. As no talk can have more than two attendees, R and S cannot both attend G. Thus, answer choice (B) can be eliminated."

Don't we know to eliminate answer choice B right away because it violates the second rule where in R cannot attend G nor H? This makes for 2 easily eliminated answer choices (B and C).
 Adam Tyson
PowerScore Staff
  • PowerScore Staff
  • Posts: 5153
  • Joined: Apr 14, 2011
|
#83524
Absolutely! Using that rule to eliminate answer B is fast and easy. Good work!

Get the most out of your LSAT Prep Plus subscription.

Analyze and track your performance with our Testing and Analytics Package.